Tanishq's roots

Algebra Level 4

Real numbers w x y z w\leq x\leq y\leq z satisfy

w + x + y + z = 12 w x + w y + w z + x y + x z + y z = 17 w x y + w x z + w y z + x y z = 114 w x y z = 216. \begin{aligned} w+x+y+z&=12\\ wx+wy+wz+xy+xz+yz&=17\\ wxy+wxz+wyz+xyz&=-114\\ wxyz&=-216. \end{aligned}

Consider the polynomial g ( u ) = w u 3 + x u 2 + y u + z . g(u)= wu^3+xu^2+yu+z.

The equation g ( u ) = 0 g(u) = 0 has roots p , q , r p, q, r . The value of p 2 + q 2 + r 2 p^2 + q^2 + r^2 can be written as a b \frac{a}{b} , where a a and b b are positive coprime integers. What is the value of a + b a+b ?

This problem is posed by Tanishq A .


The answer is 37.

This section requires Javascript.
You are seeing this because something didn't load right. We suggest you, (a) try refreshing the page, (b) enabling javascript if it is disabled on your browser and, finally, (c) loading the non-javascript version of this page . We're sorry about the hassle.

8 solutions

By Vieta's Formulas, we know that the roots of the polynomial f ( a ) = a 4 12 a 3 + 17 a 2 + 114 a 216 f(a) = a^4-12a^3+17a^2+114a-216 are w , x , y , z w,x,y,z . We note that f ( 2 ) = 0 f(2) = 0 and use this fact to help completely factor the polynomial into f ( a ) = ( a + 3 ) ( a 2 ) ( a 4 ) ( a 9 ) f(a) = (a+3)(a-2)(a-4)(a-9) . Because w x y z w\leq x \leq y \leq z , we have ( w , x , y , z ) = ( 3 , 2 , 4 , 9 ) (w,x,y,z) = (-3,2,4,9) .

\\

Now we consider g ( u ) g(u) . Again, by Vieta's Formulas, we have p + q + r = x w , p q + q r + r p = y w p+q+r = \frac{x}{w}, pq+qr+rp=\frac{y}{w} . Thus, p 2 + q 2 + r 2 = ( p + q + r ) 2 2 ( p q + q r + r p ) = p^2+q^2+r^2 = (p+q+r)^2 - 2(pq+qr+rp) = ( x w ) 2 2 y w = ( 2 3 ) 2 2 4 3 = 28 9 (\frac{x}{w})^2-2\frac{y}{w} = (\frac{2}{-3})^2 - 2\frac{4}{-3} = \frac{28}{9} , and so our answer is 28 + 9 = 37 28+9=\fbox{37} .

Moderator note:

The most common mistake made was to not remember that w x y z w \leq x \leq y \leq z , after finding the set of roots. Different ordering of these values would certainly lead to different answers.

Great job!

Alexander Borisov - 7 years, 9 months ago

Should p + q + r p+q+r be equal to x w \dfrac{-x}{w} ?

Mark Lao - 7 years, 9 months ago

Log in to reply

Yes, it should.

Jonathan Wong - 7 years, 8 months ago
Steven Hao
Sep 8, 2013

The beginning of this solution is the derivation of Vieta's formulas. Let w , x , y , z w, x, y, z be the roots of a monic 4th degree polynomial f f . Then f ( n ) = ( n w ) ( n x ) ( n y ) ( n z ) = n 4 ( w + x + y + z ) n 3 + ( w x + w y + w z + x y + x z + y z ) n 2 ( w x y + w x z + w y z + x y z ) n + w x y z = n 4 12 n 3 + 17 n 2 + 114 n 216 f(n) = (n-w)(n-x)(n-y)(n-z)\\=n^4-(w+x+y+z)n^3+(wx+wy+wz+xy+xz+yz)n^2-(wxy+wxz+wyz+xyz)n+wxyz\\=n^4-12n^3+17n^2+114n-216 We now derive similar relationships for g g . g ( u ) = w ( u p ) ( u q ) ( u r ) = w ( u 3 ( p + q + r ) u 2 + ( p + q + r ) u p q r ) = w u 3 + x u 2 + y u + z g(u) = w(u-p)(u-q)(u-r)=w(u^3-(p+q+r)u^2+(p+q+r)u-pqr)\\=wu^3+xu^2+yu+z

Now we plug things into Vieta's formulas

We factor f f using standard techniques (guess & check worked well for me) f ( n ) = ( n + 3 ) ( n 2 ) ( n 4 ) ( n 9 ) w = 3 , x = 2 , y = 4 , z = 9 f(n)=(n+3)(n-2)(n-4)(n-9)\implies w=-3, x=2, y=4, z=9

Finally, we solve the problem.

p 2 + q 2 + r 2 = ( p + q + r ) 2 2 ( p q + p r + q r ) = ( x w ) 2 2 ( y w ) = 4 9 + 8 3 = 28 9 p^2+q^2+r^2=(p+q+r)^2-2(pq+pr+qr)\\=\left(-\frac{x}{w}\right)^2-2\cdot\left(\frac{y}{w}\right)\\=\frac49+\frac83=\frac{28}9 So a + b = 28 + 9 = 37 a+b=28+9=\boxed{37}

Nice job! Yes, this is a two-step Vieta formulas problem.

Alexander Borisov - 7 years, 9 months ago

According to Vietta's formulae, w,x,y,z is the roots of the equation t 4 12 t 3 + 17 t 2 + 114 t 216 = 0 t^4-12t^3+17t^2+114t-216=0 .

Solve for t, we obtain 4 roots: -3,2,4,9.

Therefore, g ( x ) = 3 u 3 + 2 u 2 + 4 u + 9 g(x)=-3u^3+2u^2+4u+9 .

According to Vietta's formulae, p + q + r = 2 3 p+q+r=\frac{2}{3} and p q + q r + r p = 4 3 pq+qr+rp=\frac{-4}{3} .

Therefore, p 2 + q 2 + r 2 = ( p + q + r ) 2 2 ( p q + q r + r p ) = 28 9 p^2+q^2+r^2=(p+q+r)^2-2(pq+qr+rp)=\frac{28}{9} .

The answer is: 28+9=37.

Yes, this correct, great job!. You should have written a couple words on how you solved the first equation.

Alexander Borisov - 7 years, 9 months ago

w, x, y and z are seen to be the roots of the equation t 4 12 t 3 + 17 t 2 + 114 t 216 = 0 ( t + 3 ) ( t 2 ) ( t 4 ) ( t 9 ) = 0 \\ t^{4} -12t^{3} + 17t^{2}+114t -216 =0\implies (t+3)(t-2)(t-4)(t-9) =0\\ Hence w = 3 w=-3 , x = 2 x=2 , y = 4 y=4 , z = 9 z=9 . Hence p 2 + q 2 + r 2 = ( p + q + r ) 2 2 ( p q + q r + r p ) = ( x w ) 2 2 y w = 28 9 \\ p^{2} + q^{2} +r^{2} = \\ (p+q+r)^{2} - 2(pq+qr+rp) \\ = (\frac{x}{w})^{2} -\frac{2y}{w} = \frac{28}{9}

Shouvik Ganguly - 7 years, 9 months ago
Caio Pelicioni
Sep 9, 2013

Clearly, the given system of equations can lead us to a polynomial of degree 4 4 by Vieta's formulas. Using that concept, we get to:

a 4 12 a 3 + 17 a 2 + 114 a 216 = 0 a^{4} - 12a^{3} + 17a^{2} + 114a - 216 = 0

Where w, x, y and z are the roots of the polynomial.

At this point, by synthetic division we find the roots 3 , 2 , 4 , 9 -3, 2, 4, 9 .

Now, our polynomial g ( u ) g(u) has the form:

g ( u ) = 3 u 3 + 2 u 2 + 4 u + 9 g(u) = -3u^{3} + 2u^{2} + 4u + 9

By Vieta's formulas (again), we have:

p + q + r = 2 3 p + q + r = \frac{2}{3} p q + p r + r q = 4 3 pq + pr + rq = \frac{-4}{3}

In the end, we just need a little bit of manipulation:

p 2 + q 2 + r 2 = ( p + q + r ) 2 2 ( p q + p r + r q ) p^{2} + q^{2} + r^{2} = (p+q+r)^{2} - 2(pq + pr + rq) p 2 + q 2 + r 2 = ( 2 3 ) 2 2 × 4 3 p^{2} + q^{2} + r^{2} = (\frac{2}{3})^{2} -2 \times \frac{-4}{3} p 2 + q 2 + r 2 = 28 9 p^{2} + q^{2} + r^{2} = \frac{28}{9}

Thus, our answer is 28 + 9 = 37 28 + 9 = 37

Tilak Patel
Sep 14, 2013

From the given data and by using Vieta's formula, there exists a polynomial f ( a ) f(a) such that

f ( a ) = 0 f(a) = 0 for a = w , x , y , z a = w,x,y,z

Hence,

f ( a ) = a 4 12 a 3 + 17 a 2 + 114 a 216 = 0 f(a) = a^{4} - 12 a^{3} + 17a^{2}+114a - 216 = 0

On solving you will get,

f ( a ) = ( a + 3 ) ( a 2 ) ( a 4 ) ( a 9 ) = 0 f(a) = (a + 3)(a - 2)(a - 4)(a - 9) = 0

As w x y z w≤x≤y≤z

w = 3 w = -3

x = 2 x = 2

y = 4 y = 4

z = 9 z = 9

Hence, g ( u ) g(u) becomes as

g ( u ) = 3 u 3 + 2 u 2 + 4 u + 9 g(u) = -3u^{3} + 2u^{2} + 4u + 9

by using Vieta's formula

p + q + r = 2 3 p + q + r = \frac{2}{3}

p q + q r + p r = 4 3 pq + qr + pr = -\frac{4}{3}

Hence,

p 2 + q 2 + r 2 = ( p + q + r ) 2 2 ( p q + q r + p r ) p^{2} + q^{2} + r^{2} = (p + q + r)^{2} - 2(pq + qr + pr)

= ( 2 3 ) 2 2 ( 4 3 ) = ( \frac{2}{3})^{2} - 2(-\frac{4}{3})

= 28 9 = \frac{28}{9}

Hence, a + b = 28 + 9 = 37 a + b = 28 + 9 = 37

Arnav Shringi
Sep 11, 2013

By Vietta's formulae,

w,x,y,z is the roots of the equation t^4−12t^3+17t^2+114t−216=0.

By SYNTHETIC DIVISION we found the roots -3,2,4,9.

Now g(x)=−3u^3+2u^2+4u+9.

Again using Vietta's Formoula p+q+r=2/3

and pq+qr+rp=−4/3.

=> p^2+q^2+r^2=(p+q+r)^2−2(pq+qr+rp)=(2/3)^2-2[(-4/3)]=28/9.

The answer is: 28+9=37

Zheng Wei
Sep 11, 2013

by Vietta's formulae, w,x,y,z is the roots of the equation

t 4 t^{4} 12 t 3 12t^{3} + 17 t 2 17t^{2} +114t−216=0.

by SYNTHETIC DIVISION we found the roots

-3,2,4,9.

now g(x)=−3 u 3 u^{3} +2 u 2 u^{2} +4u+9.

Again using Vietta's Formoula p+q+r=2/3 and pq+qr+rp=−4/3.

=> p 2 p^{2} + q 2 q^{2} + r 2 r^{2} = ( p + q + r ) 2 (p+q+r)^{2} −2(pq+qr+rp)= ( 2 / 3 ) 2 (2/3)^{2} -2[(-4/3)]=28/9.

The answer is: 28+9=37.

Elijah Tan
Sep 11, 2013

Using Vieta's Formula, w,x,y,z are the roots of the equation. So simply by synthetic division, the roots we are are: -3, 2, 4, 9 g(x)=−3u^3+2u^2+4u+9 Using Vieta's Formula, p+q+r = 2/3 p^2+q^2+r^2 = (p+q+r)^2 − 2(pq+qr+rp) = (2/3)^2 - 2(-4/3) = 28/9. a + b = 28 + 9 = 37

0 pending reports

×

Problem Loading...

Note Loading...

Set Loading...